Active Users:214 Time:30/04/2024 08:00:24 AM
Would you consider... Duke Send a noteboard - 17/01/2016 09:06:59 AM

That it's additive and multiplied?

i.e.

Angreal might give multiplier X2 on base plus some fixed amount? This would explain how stronger channelers get more, but also explain how someone weak would get a large benefit.

Reply to message
Angreal, Sa'angreal and Moiraine at 66 - 11/01/2016 08:53:23 AM 2145 Views
Or we can choose to assume Elayne is incorrect - 11/01/2016 03:50:14 PM 1010 Views
Uhhh... - 12/01/2016 12:07:42 AM 1135 Views
Yet there are problems with either - 15/01/2016 08:52:04 PM 824 Views
Re: Yet there are problems with either - 16/01/2016 05:29:11 AM 1085 Views
Would you consider... - 17/01/2016 09:06:59 AM 948 Views
random thought on Shielding - 19/01/2016 07:34:20 PM 1007 Views
You're forgetting the other side, though. - 19/01/2016 08:19:59 PM 1069 Views
yes but it doesn't proactively do this - 19/01/2016 10:06:25 PM 932 Views
Responding to a shield doesn't require proactiveness - 20/01/2016 05:53:24 AM 826 Views
it's a visualization thing really - 20/01/2016 04:39:08 PM 882 Views
Not the crux of the debate... - 21/01/2016 03:37:40 AM 970 Views
Not really though - 21/01/2016 05:00:34 PM 737 Views
I always explained it as - 21/01/2016 09:26:35 PM 975 Views
There's not much to go on since all the shields except Berowyn's are the same - 21/01/2016 09:55:14 PM 843 Views
That's precisely my point - 21/01/2016 10:09:02 PM 986 Views
now you are speculating based on a lack of evidence - 21/01/2016 10:39:13 PM 807 Views
There's actual evidence: - 22/01/2016 06:25:25 AM 1000 Views
what's dense here is that you keep putting in quotes that don't support your position - 22/01/2016 03:28:16 PM 1119 Views
Whoa.. - 22/01/2016 04:24:19 PM 1068 Views
Not at all - 22/01/2016 05:03:50 PM 1012 Views
Wonderful - 22/01/2016 06:30:35 PM 987 Views
yup that's my argument. that and you're a dick so I'm done with this - 22/01/2016 06:46:23 PM 806 Views
Re: yup that's my argument. that and you're a dick so I'm done with this - 23/01/2016 02:35:33 PM 1104 Views
Petty much *NM* - 24/01/2016 02:50:32 PM 446 Views
Hmmm.... - 23/01/2016 03:06:15 PM 1055 Views
Let me clear this up - 25/01/2016 04:19:51 PM 1192 Views
Some more quotes - 25/01/2016 05:10:51 PM 955 Views
none of which I've denied or tried to prove otherwise - 25/01/2016 07:19:48 PM 1244 Views
Re: none of which I've denied or tried to prove otherwise - 26/01/2016 03:45:52 AM 1017 Views
Re: none of which I've denied or tried to prove otherwise - 26/01/2016 09:00:55 AM 1164 Views
Re: none of which I've denied or tried to prove otherwise - 26/01/2016 10:39:49 AM 950 Views
Oh well then I agree with you - 26/01/2016 08:50:55 AM 1065 Views
thanks - 26/01/2016 04:26:46 PM 1221 Views
Yes, but that's not what I'm arguing... - 16/01/2016 08:56:15 AM 788 Views
But additive doesn't explain buffers and being able to overdraw - 16/01/2016 03:02:33 PM 820 Views
Don't those two facts explain each other? - 16/01/2016 03:24:44 PM 860 Views
It actually seems counterintuitive to me - 19/01/2016 07:15:37 PM 803 Views
Simple - 19/01/2016 08:21:11 PM 926 Views
Not at all - 19/01/2016 10:17:39 PM 713 Views
Huh? - 20/01/2016 06:01:04 AM 926 Views
agree to disagree I suppose ... I don't see it this way *NM* - 20/01/2016 04:41:16 PM 501 Views
I didn't sense disagreement so much as confusion over my position. *NM* - 21/01/2016 12:01:16 AM 480 Views
Re: I didn't sense disagreement so much as confusion over my position. - 21/01/2016 02:07:21 AM 829 Views
Re: I didn't sense disagreement so much as confusion over my position. - 21/01/2016 03:32:59 AM 848 Views
I don't necessarily think that's true - 21/01/2016 05:07:40 PM 921 Views
I don't see how magnifiers solve this - 21/01/2016 10:01:17 PM 908 Views
Re: I don't see how magnifiers solve this - 21/01/2016 10:16:16 PM 848 Views
Uhhh... - 22/01/2016 06:51:11 AM 996 Views
Funny, I just saw this post - 17/09/2016 11:13:09 PM 715 Views
The very first chapter (the Prologue) disproves this - 03/10/2016 06:56:28 AM 790 Views
No it doesn't - 05/10/2016 12:47:03 AM 685 Views
Re: Don't those two facts explain each other? - 08/10/2016 05:06:53 AM 585 Views
Re: Yes, but that's not what I'm arguing... - 08/10/2016 04:52:06 AM 802 Views

Reply to Message